Difference between revisions of "2010 AIME II Problems/Problem 1"

m (Solution)
m
Line 5: Line 5:
 
If we include all the even digits for the greatest integer multiple, we find that it is impossible for it to be divisible by <math>3</math>, therefore by <math>36</math> as well.
 
If we include all the even digits for the greatest integer multiple, we find that it is impossible for it to be divisible by <math>3</math>, therefore by <math>36</math> as well.
 
The next logical try would be <math>8640</math>, which happens to be divisible by <math>36</math>. Thus <math>N = 8640 \pmod {1000} = \boxed{640}</math>
 
The next logical try would be <math>8640</math>, which happens to be divisible by <math>36</math>. Thus <math>N = 8640 \pmod {1000} = \boxed{640}</math>
 +
 +
== See also ==
 +
{{AIME box|year=2010|before=First Problem|num-a=2|n=II}}
 +
 +
[[Category:Introductory Number Theory Problems]]

Revision as of 17:14, 3 April 2010

Problem

Let $N$ be the greatest integer multiple of $36$ all of whose digits are even and no two of whose digits are the same. Find the remainder when $N$ is divided by $1000$.

Solution

If we include all the even digits for the greatest integer multiple, we find that it is impossible for it to be divisible by $3$, therefore by $36$ as well. The next logical try would be $8640$, which happens to be divisible by $36$. Thus $N = 8640 \pmod {1000} = \boxed{640}$

See also

2010 AIME II (ProblemsAnswer KeyResources)
Preceded by
First Problem
Followed by
Problem 2
1 2 3 4 5 6 7 8 9 10 11 12 13 14 15
All AIME Problems and Solutions